1answer.
Ask question
Login Signup
Ask question
All categories
  • English
  • Mathematics
  • Social Studies
  • Business
  • History
  • Health
  • Geography
  • Biology
  • Physics
  • Chemistry
  • Computers and Technology
  • Arts
  • World Languages
  • Spanish
  • French
  • German
  • Advanced Placement (AP)
  • SAT
  • Medicine
  • Law
  • Engineering
vovikov84 [41]
1 year ago
8

A loop rests in the plane of a page of textbook while a magnetic field is directed into the page. A clockwise current is induced

.
Physics
1 answer:
weqwewe [10]1 year ago
7 0

(b) when the magnetic field gets stronger.

correct option :

When a loop rest in the plane of the page and magnetic field is directed into the page then we will apply here right hand thumb rule in which thumb represent the direction of magnetic field and curling fingers represent direction of induced current. So, in this case the direction of magnetic field is into the page and curling fingers shows induced current in clockwise direction.

  Thus, option (b) is correct.

Incorrect options:

   option (A) is incorrect because when magnetic field will change  

     according to flux.

   option (C) is incorrect because field is stronger so it does not matter

     size of loop increases or decreases.

   option (D) is incorrect because it moves any other side the field will be

     stronger.

Learn more about magnetic field induction here:

 brainly.com/question/13610297

   #SPJ4

[Your question is incomplete, but most probably your full question was-Check all that apply:

A. when the magnetic field is tilted so it is no longer perpendicular to the page

B. when the magnetic field gets stronger

C. when the size of the loop decreases

D. when the loop is moved sideways across the page]

You might be interested in
Why do smaller endotherms require more energy per unit of mass than larger endotherms?
Naily [24]
They expend more oxygen. Littler endotherms lose warmth to the earth proportionately speedier than huge endotherms: less warm mass, protecting layers in littler creatures are less successful by dint of being more slender, and more prominent surface region to volume proportion implies snappier radiation of warmth
4 0
2 years ago
Titan Tommy and the Test Tubes at a nightclub this weekend. The lead instrumentalist uses a test tube (closed-end air column) wi
Westkost [7]

Hello , it' s 5 but i am not sure Explanation:

6 0
2 years ago
If a 20 kg green fish swimming at 2 m/s swallows a 1 kg orange fish at rest, in what direction, and how fast
krok68 [10]

Answer: 1.9 m/s

Explanation:

The question should be:

If a 20 kg green fish swimming at 2 m/s swallows a 1 kg orange fish at rest, in what direction, and how fast  will the green fish swim after eating the orange fish?

Ok, here we have conservation of momentum.

At the beginning, the total momentum is equal to the sum between the momentum of the green fish and the momentum of the orange fish.

Where the momentum is written as:

P = m*v

m = mass

v = velocity.

The momentum of the green fish is:

Pg = 20kg*2m/s = 40 kg*m/s.

The momentum of the orange fish is:

Po = 1kg*0m/s = 0

The total initial momentum is:

Pi = Pg + Po = 40 kg*m/s.

After the green fish eats the orange fish, we do not have an orange fish anymore, and the mass of the green fish will be equal to it's initial mass, plus the mass of the fish that it ate, this will be:

M = 20kg + 1kg = 21kg.

Then the momentum will be:

Pf = 21kg*V

Where V is the final velocity.

For conservation of momentum, the initial momentum is equal to the final momentum, then:

Pi = Pf

40 kg*m/s = 21kg*V

(40/21) m/s = 1.9 m/s = V

The fish's final velocity is 1.9 m/s

5 0
2 years ago
PLEASE HELP ME ASAP!!!!!<br> WHAT AM I SUPPOSED TO DO??
igomit [66]
Write what you can improve in your fitness test
3 0
2 years ago
Read 2 more answers
local AM radio station broadcasts at a frequency of 685.9 kHz. Calculate the wavelength at which they are broadcasting. Waveleng
natta225 [31]

Answer:

λ = 437 m.

Explanation:

  • Since a radio wave is a electromagnetic type of wave, it propagates approximately at the same speed of light in vacuum, 3*10⁸ m/s.
  • As in any wave, there exist a fixed relationship between the propagation speed, the wavelength and the frequency of the radio wave, as follows:

       c = \lambda* f (1)

  • Replacing by the values of the speed and the frequency, and solving for the wavelength λ, we get:

       \lambda = \frac{c}{f} =\frac{3e8 m/s}{685.9e31/s} =437 m  (2)

8 0
2 years ago
Other questions:
  • Please help!!!! due today!!
    6·1 answer
  • What is the order of magnitude of the gravitational force between two 1.0 kilogram charges that are positioned 1.0 meter apart?
    5·1 answer
  • When he sees teachers encouraging other children to wait in the cafeteria until the first bell rings, Ian follows them. What typ
    11·2 answers
  • A vector has a length of 10 and rises
    11·1 answer
  • Please need help on this
    8·1 answer
  • How do lone pairs of electrons affect the bond angle differently than electrons shared in a bond?
    9·1 answer
  • For variables control, a circuit voltage will be measured using a sample of five circuits. The past average voltage for samples
    7·1 answer
  • In a thunderstorm at 32.0°C, Reginald sees a bolt of lightning and hears the thunderclap 2.00s later. How far from Reginald did
    8·1 answer
  • When is the particle in figure (a) speeding up? (enter your answer using interval notation.)
    10·1 answer
  • If 100g of an isotope undergoes two half-lives, how many grams will be remaining?
    14·1 answer
Add answer
Login
Not registered? Fast signup
Signup
Login Signup
Ask question!